Tricircles

Geometry Level 2

I have 3 unit circles. Two of them are externally tangent to each other, and the third one passes through the tangent point, cutting two symmetrical areas from the two circles, as shown above. What is the shaded area?

2 2 π 1 \pi-1 2 π 4 2\pi-4 6 π 6-\pi

This section requires Javascript.
You are seeing this because something didn't load right. We suggest you, (a) try refreshing the page, (b) enabling javascript if it is disabled on your browser and, finally, (c) loading the non-javascript version of this page . We're sorry about the hassle.

8 solutions

David Vreken
Jan 18, 2018

The shaded area can be split up into 4 4 regions with right angles and unit legs, and then rearranged into a 2 2 by 1 1 rectangle, as shown below.

Therefore, the area of the shaded region is the same as the area of the 2 2 by 1 1 rectangle, which is 2 \boxed{2} .

I believe the question is not worded quite right. The third circle can pass through the tangent but necessarily remain symmetrical around it. The solution requires it also be symmetrical.

Timothy Miller - 3 years, 4 months ago

Log in to reply

I agree that the question should include something about its assumed symmetry that is in the picture. If it weren't symmetrical, an extreme case would be that the third circle completely overlaps the second circle (still making it pass through the tangent point) making the shaded area equal to zero.

David Vreken - 3 years, 4 months ago

Log in to reply

Yes. It should be stated that the third circle's center is equidistant from the centers of the tangent circles

Norbert Tureks - 3 years, 4 months ago

Log in to reply

@Norbert Tureks That's really true

Riya Verma - 3 years, 4 months ago

If you read carefully the text of the problem, then you can find: "...cutting two symmetrical areas from the two circles..." So, please read before writing.

Angel Krastev - 3 years, 4 months ago

Log in to reply

The problem must have been updated (and improved). The phrase "cutting two symmetrical areas from the two circles" was not there a few days ago.

David Vreken - 3 years, 4 months ago

You have made assumptions based on a drawing. Symmetry is not a measure. You must prove that the "crosshairs" in the shaded circle are right angles. Symmetry does not imply angular equality. You assume that the horizontal line is exactly equal to the vertical line. This is not known.

Brent R Jones - 3 years, 4 months ago

Log in to reply

Proved more rigorously, let O O be the center of the top circle, P P be the center of the left circle, Q Q be the center of the right circle, A A be the left intersection of circles O O and P P , B B be the right intersection of circles O O and Q Q , and C C be the intersection of all three circles. Since all three circles are unit circles, O A = O B = O C = P A = P C = Q C = Q B = 1 OA = OB = OC = PA = PC = QC = QB = 1 . Also, since circle O O cuts two symmetrical areas from the two circles, A C B C AC \cong BC . Then A P C A C O B C O B C Q \triangle APC \cong \triangle ACO \cong \triangle BCO \cong \triangle BCQ by SSS, and A P C A C O B C O B C Q \angle APC \cong \angle ACO \cong \angle BCO \cong \angle BCQ by corresponding parts. Since circle P P and circle Q Q are externally tangent to each other, P C Q \angle PCQ is a straight line equaling 180 ° 180° , meaning m A P C = m A C O = m B C O = m B C Q = 45 ° m\angle APC = m\angle ACO = m\angle BCO = m\angle BCQ = 45° . Since O A C \triangle OAC is an isosceles triangle, m C A O = 45 ° m\angle CAO = 45° and using the angle sum of A O C \triangle AOC , m A O C = 90 ° m\angle AOC = 90° . A similar argument will show that m B O C = 90 ° m\angle BOC = 90° .

Therefore, the "crosshairs" in the shaded region are right angles, and the horizontal line segment in the shaded region is exactly equal to the vertical line segment in the shaded region.

David Vreken - 3 years, 4 months ago

Log in to reply

haha! Get reken by vreken

lil pump - 3 years, 4 months ago

Are you assuming r=1 or the answer would be 2r?

Andre Lopes - 3 years, 4 months ago

Log in to reply

I think because they are unit circles, it can be assumed that there radii are equal to one.

Syed Ahmad Raza - 3 years, 4 months ago

Log in to reply

Yes unit circles have radius 1

erica phillips - 3 years, 4 months ago

Elegant solution... And yes the question does imply symmetry

Calogero Matteliano - 3 years, 4 months ago

great solution and awesome visualization!!

erica phillips - 3 years, 4 months ago

Wow!!! Elegant approach.

A Former Brilliant Member - 3 years, 4 months ago

Beautiful!!!! Geometry using geometry.....

Mausam Mathur - 3 years, 4 months ago

Question not worded properly

Pedro Gonzalez - 3 years, 4 months ago

Cut and then put the parts together

Crystal Wang - 3 years, 4 months ago
Marta Reece
Jan 16, 2018

The area of the shape is equal to the area of a square inscribed in a unit circle, that is a square with side 2 \sqrt2 and area 2 \boxed2 .

How do you conclude that the length of the side of the square is √2

Pranav L - 3 years, 4 months ago

Log in to reply

Because the square can be divided into 4 right triangles, each with 2 sides adjacent to a right angle with length 1. The sides (hypotenuses/hypoteni?) marked with 1,2,3,4 above would be the square root of the sum of the squares of those adjacent sides.

Jeremiah Cassidy - 3 years, 4 months ago

Since it is a unit circle we know the radius is 1. Divide the square into four equal triangles that each have the radii as two sides and an edge of the square as a third side. Then use pythagorean theorem. If 1^2 + 1^2 = X^2 then X = square root of 2

Jesse Boldt - 3 years, 4 months ago

I really don't see where I went wrong. I also worked with segments. 4 segments x (PI*r^2/4 + 1/2 bh)= PI -2 That's for quarter of a circle. But I get the same answer if I use 2x(half circle area - triangle with base 2 and height 1). I must be going crazy.....

Ayan Booyens - 3 years, 4 months ago

Rather odd... Finding an integer as the area of a region clearly dependent on Pi.

T. Garza

Tomas Garza - 3 years, 4 months ago

At first it's hard to understand why quadrangle is a square,but after proving it is, this way of solving problems becomes quite smart

Crystal Wang - 3 years, 4 months ago
Tousif Ahmad
Jan 30, 2018

area of region "d" = area of square (d+e) - area of circular sector(e) = 1 π 4 1-\frac{\pi}{4}

therefore area of a+b+c+d = 2 x area of b+d = 2 × ( π 4 + ( 1 π 4 ) ) = 2 2 \times ( \frac{\pi}{4} + (1-\frac{\pi}{4}) )= 2

This seems less intuitive and more like a proof. Upvoting this solution.

James Tucker - 3 years, 4 months ago

Log in to reply

thats because intuitive solutions were already available ..

Tousif Ahmad - 3 years, 4 months ago

I'm using the area equation and I got 2 r^2 as the answer....

darren vincent - 3 years, 4 months ago

I was misled by the term "unit circles." I thought this meant circles of unit area, not of unit radius. I think the term "circles of unit radius" would have been a better one to use. (I'm not saying I'd have got the right answer though!!!!)

Brian Turton - 3 years, 4 months ago

Might be pretty clear for most, but I took a while to wrap my head around how the (d+e) was area enclosed by a square. The line segments are all 1 unit in length and one of the angles can be proved to be 90 degrees as the angle between the radius and tangent is 90.

Sahil Shaheen - 3 years, 3 months ago
Adam Dougall
Jan 31, 2018

The question as emailed does not have a unique solution as it omits the crucial phrase "cutting two symmetrical areas from the two circles".

it does say 2 symmetrical areas

Bruce Dee - 3 years, 4 months ago
Alkis Piskas
Feb 1, 2018

As the problem is described, the following case also applies: which gives a total different result. So the folowing should be added to the problem description: The 3d circle must "cut the two other circles into 90-degree segments" (or "have the same radius with the two other circles"). In this case, the area of a segment is r^2 (π-2)/4 (result of area of a 90-degree segment). Two of them make a r^2 (π-2)/2, which form a "leaf". Subtracting two "leaves" from the total ara circle, we get: r^2 π - r^2 (π-2) = r^2 π - r^2 π + 2 r^2 = 2 r^2 = 2

The problem stated 3 unit circles. Doesn't that mean all three circles are the same size with an r=1?

Neal Kaplan - 3 years, 4 months ago

Log in to reply

Yes, that is true, it is obvious as it states "3 unit circles".

Syed Hamza Khalid - 3 years, 4 months ago

why is 2r^2 = 2? do we replace r with 1?

darren vincent - 3 years, 4 months ago

Log in to reply

Yep. U may check my answer. it has the proof

Syed Hamza Khalid - 3 years, 4 months ago

This was stated in the problem: "3 unit circles"

That means that all 3 circles have radius = 1

Your 3 circles have different sizes.

Lupuleasa I - 3 years, 4 months ago

Log in to reply

Yeah. True.

Syed Hamza Khalid - 3 years, 4 months ago

Using the equation of a circle and the integral of the differece of two functions bounding a region gives directly the area of that region. Choosing the origine of the x- and y-axis to be the point of intersection of the two tangent circles and using the symetrical aspect of the problem one finds:

A r e a = 2 0 1 ( 1 + 1 x 2 1 ( x 1 ) 2 ) d x = 2 Area = 2 \cdot \int_0^1(1+\sqrt{1-x^2}-\sqrt{1-(x-1)^2})\,\mathrm{d}x = 2

Would you be able to provide an explanation using a double integral with r dr d(theta)? I attempted it but I couldn't find the right areas of integration for the radius (which I tried to let it go from 1 to cos^2(theta) + sin^2(theta) but to no avail).

Trevor Ohara-Oates - 3 years, 4 months ago

Log in to reply

...(which I tried to let it go from 1 to cos^2(theta) + sin^2(theta) but to no avail).

cos 2 θ + sin 2 θ = 1 \cos^2 \theta + \sin^2 \theta = 1 , which means your integral evaluates to 0. This suggests that your limits are wrong in the first place.

Pi Han Goh - 3 years, 4 months ago

Log in to reply

Ahh, a stupid mistake... I tried it again but I'm unsure of the interval in which I should let r vary through. I am setting my origin at the center of the purple circle and I know r has to be from 1 (starting when theta = 0) along the path of the outer circumference of the two intersecting circles, my problem is simply trying to find this equation for when the circles intersect. Any help would be much appreciated!

Trevor Ohara-Oates - 3 years, 4 months ago

Log in to reply

@Trevor Ohara-Oates I think you're just finding an overcomplicated approach to a simple question. I don't think it's really fruitful to think about double integrals here.

Pi Han Goh - 3 years, 4 months ago
Ricardo Raymond
Feb 1, 2018

Take a quarter circle. Area of which is pie/4. Area of triangle in this quarter circle is .5×1×1×sin 90% = .5 The are of the chord = pie/4 - .5 There are 4 of these in the circle. So multiply by 4 = pie -2. Area of full circle = pie. Hence shaded area is pie-(pie-2) = 2.

Syed Hamza Khalid
Jan 31, 2018

Area of this shape using any length for radius is equal to:

A = 2 r 2 \large \color{#3D99F6} \text{A = 2}r^2

Hence, inputting "1" gives 2


How to get this formula:

As you can see in the above image, the red lines represent the radius r \large {r} ; I have made a square too. Clearly to understand that there is a quarter of a circle whose area is π r 2 4 \frac{\pi r^2}{4} . Hence we can say that:

Area of the shape = Area of semi-circle + 2 times (the area of the square - the area of the quadrant) = π r 2 2 + 2 ( r 2 π r 2 4 ) = π r 2 2 + 2 r 2 π r 2 2 = 2 r 2 \large \color{#20A900} \text{Area of the shape = Area of semi-circle + 2 times (the area of the square - the area of the quadrant) }\\= \large \color{#20A900} \frac{\pi r^2}{2} + 2(r^2 - \frac{\pi r^2}{4}) \\ \large \color{#20A900}= \frac{\pi r^2}{2} + 2r^2 - \frac{\pi r^2}{2} \\ \large \color{#624F41} = 2r^2

How do you get this???

Riya Verma - 3 years, 4 months ago

Log in to reply

Sorry I will prove it soon.... I edited the answer

Syed Hamza Khalid - 3 years, 4 months ago

0 pending reports

×

Problem Loading...

Note Loading...

Set Loading...